A bond with a par value of $1,000 and an annual coupon has a yield to maturity of 5.60% and a current price of $975. If the bond has 18 years to maturity, what is its current yield?

Answers

Answer 1

Answer:

Current Yield is 5.74%

Explanation:

Current yield is the ratio of coupon payment of a bond to its current market price.  It is calculated by using coupon payment and the current market value of the bond.

Coupon Payment = $1,000 x 5.6% = $56

Current market price = $975

Formula for Current yield is as follow

Current Yield = Annual Coupon Payment / Current Market Price

Current Yield = $56 / $975

Current Yield = 0.0574% = 5.74%


Related Questions

FIFO Perpetual Inventory

The beginning inventory at Dunne Co. and data on purchases and sales for a three-month period ending June 30 are as follows:

Date Transaction Number
of Units Per Unit Total
Apr. 3 Inventory 48 $150 $7,200
8 Purchase 96 180 17,280
11 Sale 64 500 32,000
30 Sale 40 500 20,000
May 8 Purchase 80 200 16,000
10 Sale 48 500 24,000
19 Sale 24 500 12,000
28 Purchase 80 220 17,600
June 5 Sale 48 525 25,200
16 Sale 64 525 33,600
21 Purchase 144 240 34,560
28 Sale 72 525 37,800
Required:

1. Record the inventory, purchases, and cost of merchandise sold data in a perpetual inventory record similar to the one illustrated in Exhibit 3, using the first-in, first-out method. Under FIFO, if units are in inventory at two different costs, enter the units with the LOWER unit cost first in the Cost of Goods Sold Unit Cost column and in the Inventory Unit Cost column.

Dunne Co.
Schedule of Cost of Goods Sold
FIFO Method
For the Three Months Ended June 30
Purchases Cost of Goods Sold Inventory
Date Quantity Unit Cost Total Cost Quantity Unit Cost Total Cost Quantity Unit Cost Total Cost
Apr. 3 $ $
Apr. 8 $ $
Apr. 11 $ $
Apr. 30
May 8
May 10
May 19
May 28
June 5
June 16
June 21
June 28
June 30 Balances $ $
2. Determine the total sales and the total cost of goods sold for the period. Journalize the entries in the sales and cost of goods sold accounts. Assume that all sales were on account.

Record sale
Record cost
3. Determine the gross profit from sales for the period.
$

4. Determine the ending inventory cost as of June 30.
$

5. Based upon the preceding data, would you expect the ending inventory using the last-in, first-out method to be higher or lower?

Answers

Answer:

Dunne Co.

Schedule of Cost of Goods Sold

FIFO Method

For the Three Months Ended June 30

                                          Purchases      Cost of Goods Sold Inventory

Date            Description    Quantity    Unit Cost       Total Cost   Sales

Apr. 3          Inventory         48                $150             $7,200

Apr. 8          Purchase         96                  180              17,280

Apr. 11         Sale                           64        500                             32,000

Apr. 30       Sale                           40        500                             20,000

May 8         Purchase         80                 200             16,000

May 10       Sale                           48        500                             24,000

May 19       Sale                           24        500                             12,000

May 28      Purchase         80                  220            17,600

June 5       Sale                          48         525                             25,200

June 16      Sale                         64         525                             33,600

June 21      Purchase      144                  240            34,560

June 28     Sale                         72         525                             37,800

June 30     Total            448   360                         $92,640 $184,600

June 30     Balances       88                 $240          $21,120      

2. Determination of total sales and cost of goods sold and Journal Entries:

Debit Accounts Receivable $184,600

Credit Sales Revenue $184,600

To record the sales of goods on account for the period.

Debit Cost of Goods Sold $92,640

Credit Inventory $92,640

To record the cost of goods sold for the period.

3. Income Statement for determining the gross profit:

Sales Revenue       $184,600

Cost of goods sold $92,640

Gross profit             $91,960

4. Determination of the ending inventory cost of June 30:

Ending Inventory units = 88

Cost per unit (FIFO) = $240

Total =                     $21,120

5. The ending inventory would be lower if the ending inventory was valued using the Last-in, First-out (LIFO) method.  The purchase price was increasing instead.  Using LIFO means that ending inventory would be valued at the cost of the purchases in earlier months because of the assumption with LIFO that goods sold are from the last purchases instead of the earlier purchases.

Explanation:

Knowing she has sold 5,000 pairs, assume the company wants to launch a Black Friday promotion, where she would discount her shoes by 10%. How many more shoes would she have to sell to justify this promotion

Answers

Revenue: $500,000

Shoes: $250,000

Shoe boxes: $1,000

Advertising: $500

Rent: $1,000

Depreciation: $25

Knowing she has sold 5,000 pairs, assume the company wants to launch a Black Friday promotion, where she would discount her shoes by 10%. How many more shoes would she have to sell to justify this promotion?

A. 25.13% more shoes

B. 20.08% more shoes

C. None of the above, but I could calculate this with the information I am given.

D. None of the above, I cannot calculate this with the information I am given.

Answer:

Option A. 25.13% more shoes

Explanation:

Cost Benefit analysis would be useful here to acknowledge what percentage of shoe sales is required to justify the promotion.

The Benefit drawn before 10% promotion proposal:

Revenue:                           $500,000

Shoes:                               ($250,000)

Shoe boxes:                         ($1,000)

Advertising:                           ($500)

Rent:                                     ($1,000)

Depreciation:                          ($25)

Profit                                    $247,475

The Benefit drawn before 10% promotion proposal:

Revenue:                           $450,000

Shoes:                               ($250,000)

Shoe boxes:                        ($1,000)

Advertising:                          ($500)

Rent:                                    ($1,000)

Depreciation:                         ($25)

Profit                                   $197,475

Now we can calculate how much additional sales must be required to justify the promotion.

Sales Increase Required = (Initial Profit - Before Promotion) / Profit After Promotion

Sales Increase Required = ($247,475  - $197,475) / $197,475

Sales Increase Required = 25.31% which is close to option 1, hence Option 1 is correct here.

Promotion is termed as the activity that involves the spreading or publicizing of information regarding the products and services. It is a part of marketing that involves publicity and public relations between the customers.  

The correct option is A. 25.13% more shoes

Cost Benefit analysis would be useful here to acknowledge what percentage of shoe sales is required to justify the promotion.

The Benefit drew before 10% promotion proposal:

Revenue:                           $500,000

Shoes:                               ($250,000)

Shoe boxes:                         ($1,000)

Advertising:                           ($500)

Rent:                                     ($1,000)

Depreciation:                          ($25)

Profit                                    $247,475

The Benefit drew before 10% promotion proposal:

Revenue:                           $450,000

Shoes:                               ($250,000)

Shoe boxes:                        ($1,000)

Advertising:                          ($500)

Rent:                                    ($1,000)

Depreciation:                         ($25)

Profit                                   $197,475

Now we can calculate how much additional sales must be required to justify the promotion.

Sales Increase Required = [tex]\frac{\text{Initial Profit - Before Promotion}}{\text{Profit After Promotion}}[/tex]

Sales Increase Required = [tex]\frac{\$247,475-\$197,475}{\$197,475}[/tex]

Sales Increase Required = 25.31% which is close to option 1, hence Option 1 is correct here.

To know more about the promotional activities, refer to the link below:

https://brainly.com/question/15869831

Suppose that Dunkin Donuts reduces the price of its regular coffee from $2 to $1 per cup, and as a result, the quantity sold per day increased from 10 to 40. Over this price range, the price elasticity of demand for Dunkin Donuts’ regular coffee is:

Answers

Answer:

PED = -6

Explanation:

The PED or price elasticity of demand for a product measures the responsiveness of a product's demand to the changes in the price of the product. The PED is calculated as follows,

PED = % change in Quantity demanded / % change in price

PED = [(40 - 10) / 10]  /  [(1 - 2) / 2]

PED = -6

A PED of -6 represents that quantity demanded is highly price elastic and a negative sign means that it is a normal good.

Burpee Company sells seeds to garden stores. Sales are expected to be $2,038,635 in January, $2,581,891 in February and $2,913,307 in March. Burpee sets their prices so that they earn an average 32% gross profit on sales revenue. What is budgeted cost of goods sold for the first quarter (January, February and March)?

Answers

Answer:

Total COGS= $5,123,006.44

Explanation:

Giving the following information:

Sales:

January= $2,038,635

February= $2,581,891

March= $2,913,307

Burpee sets their prices so that they earn an average 32% gross profit on sales revenue.

We need to calculate the cost of goods sold:

January= 2,038,635*0.68= 1,386,271.8

February= 2,581,891*0.68= 1,755,685.88

March= 2,913,307*0.68= 1,981,048.76

Total COGS= $5,123,006.44

Beyer Company is considering the purchase of an asset for $190,000. It is expected to produce the following net cash flows. The cash flows occur evenly within each year.
Year 1 Year 2 Year 3 Year 4 Year 5 Total
Net cash flows $50,000 $31,000 $60,000 $140,000 $30,000 $311,000
Compute the payback period for this investment.

Answers

Answer:

Pay back period =3 years 4 months

Explanation:

The payback period is the estimated length of time it takes cash inflow from a project to recoup the cash outflow.  

The payback period uses cash flows and not profit.

The payback period can be determined by accumulation the cash inflow consecutively to ascertain the length of time it will take the sum to equate the initial cost.

This will be done as follows:

The sum of the cash in flows for the first three years would equal

50,000 + $31,000 + $60,000 = 141,000

The balance required to equate 190,000 would be

balance = 190,000 - 140,000 = 50,000

Pay back period = 3 years + (50,000/140,000)× 12 months

                            = 3 years 4 months

Pay back period =3 years 4 months

Pear Corporation is considering Alternative A and Alternative B. Costs associated with the alternatives are listed below:
Alternative A Alternative B
Materials costs $ 40,000 $ 56,000
Processing costs $ 37,000 $ 37,000
Equipment rental $ 13,000 $ 13,000
Occupancy costs $ 15,000 $ 22,000
Are the materials costs and processing costs relevant in the choice between alternatives A and B?
Multiple Choice
A) Only processing costs are relevant
B) Only materials costs are relevant
C) Both materials costs and processing costs are relevant
D) Neither materials costs nor processing costs are relevant

Answers

Answer: B) Only materials costs are relevant

Explanation:

When choosing between alternatives, the main decider is the difference in costs. The costs that are different are the ones to decide whether a company takes on a project as it will signal the financial viability of a project.

In both alternatives, the Processing costs remain at $37,000 therefore the alternative chosen is irrelevant to these costs as they will be incurred regardless of the company's choice. They are therefore not to be considered.

Material costs on the other hand vary by the alternatives and so should be considered.

Many managers describe performance appraisal as the responsibility that they like least. Why is this so? What could be done to improve the situation?

Answers

Answer:

Many managers describe performance appraisal as the responsibility that they like least. Why is this so?

it might be so because managers may feel that performance appraisal is not as productive as other activities, or because they lack the personal skills, or the motivation, to engage in that activity.

What could be done to improve the situation?

Managers should be taught that performance appraisal can be a very effective and productive method for the firm. When workers are praised for their work (when they deserve it), they are likely to be happier in the workplace, and it has been shown by countless studies that happier workers are also more productive.

Sonic Inc. manufactures two models of speakers, Rumble and Thunder. Based on the following production and sales data for June, prepare (a) a sales budget and (b) a production budget: Rumble Thunder Estimated inventory (units), June 1 750 300 Desired inventory (units), June 30 500 250 Expected sales volume (units): Midwest Region 12,000 3,500 South Region 14,000 4,000 Unit sales price $60 $90 a. Prepare a sales budget.

Answers

Answer: please see explanation column

Explanation:

                                                        Rumble Thunder

Estimated inventory (units), June 1 750 300

Desired inventory (units), June 30 500 250

Expected sales volume (units):

Midwest Region                           12,000 3,500

South Region                                14,000 4,000

Unit sales price                                 $60 $90

a)               Sonic Inc.  Sales Budget  for June

                        Unit Sales Vol Unit Selling price    Total Sales

Model Rumble:    

Midwest Region   12000      60         $720,000

South Region   14000      60          $840,000

Total   1,560,000

Model Thunder:    

Midwest Region 3500       $90           $315,000

South Region         4000       $90            $360,000

Total   $675,000

Total revenue from sales 1,560,000  + $675,000 =$2,235,000

B)               Sonic Inc.  Production budget for June

                                              Units Model Rumble Units Model Thunder

Expected units to be sold  26000                  7500

Add: Desired ending inventory   + 500                  +  250

Total units required                   26500                    7750

Less: Beginning inventory            - 750                     - 300

Total units to be produced  $25750                   $ 7450

Calculation :

Expected units to be sold =12,000  + 14,000 = $26,000

                                               3,500 + 4,000 = $7,500

Total units required=Expected units to be sold+ Desired ending inventory

26000 +500 =$26,500

7,500 +250= $7,750

Sheridan Company prepared a 2019 budget for 150000 units of product. Actual production in 2019 was 175000 units. To be most useful, what amounts should a performance report for this company compare

Answers

Answer:

The actual results for 175,000 units with a new budget for 175,000 units.

Explanation:

To be more useful, actual results should be compared with budgeted amounts of actual production.

The actual results for 175,000 units should be compare with a new budget for 175,000 units

The Cutting Department at Blanc Company had beginning work in process inventory of 4,000 units, transferred out 9,000 units, and had 2,000 units in ending work in process inventory. The number of units started into production by the Cutting Department during the month is

Answers

Answer:

The number of units started into production is 7,000.

Explanation:

Number of units started into production = Units transferred out + units of ending work in process - units of beginning work in process

                                                                   = 9,000 + 2,000 - 4,000

                                                                   = 7,000

You are planning to save for retirement over the next 25 years. To do this, you will invest $880 per month in a stock account and $480 per month in a bond account. The return of the stock account is expected to be an APR of 10.8 percent, and the bond account will earn an APR of 6.8 percent. When you retire, you will combine your money into an account with an APR of 7.8 percent. All interest rates are compounded monthly. How much can you withdraw each month from your account assuming a withdrawal period of 20 years

Answers

Answer:

$14,143.86 can be withdrawn each month from the account for 20 years.

Explanation:

To determine this, the first step is to use the formula for calculating the future value (FV) of ordinary annuity to calculate the FV of both stock and bond as follows:

Calculation of Future Value of Stock

FVs = M × {[(1 + r)^n - 1] ÷ r} ................................. (1)

Where,

FVs = Future value of the amount invested in stock after 25 years =?

M = Monthly investment = $880

r = Monthly interest rate = 10.8% ÷ 12 = 0.9%, or 0.009

n = number of months = 25 years × 12 months = 300

Substituting the values into equation (1), we have:

FVs = $880 × {[(1 + 0.009)^360 - 1] ÷ 0.009}

FVs = $880 × 1,522.3445923122

FVs = $1,339,663.24

Calculation of Future Value of Bond

FVd = M × {[(1 + r)^n - 1] ÷ r} ................................. (1)

Where,

FVd = Future value of the amount invested in bond after 25 years =?

M = Monthly investment = $480

r = Monthly interest rate = 6.8% ÷ 12 = 0.566666666666667%, or 0.00566666666666667

n = number of months = 25 years × 12 months = 300

Substituting the values into equation (1), we have:

FVd = $480 × {[(1 + 0.00566666666666667)^300 - 1] ÷ 0.00566666666666667}

FVd = $480 × 784.895879465925

FVd = $376,750.02

Calculation of the amount that can be withdrawn monthly for 20 years

To calculate this, the formula for calculating the present value of an ordinary annuity is used as follows:

PV = P × [{1 - [1 ÷ (1+r)]^n} ÷ r] …………………………………. (3)

Where;

PV = Combined present values of stock and bond investments after retirement = FVs + FVb = $1,339,663.24 + $376,750.02 = $1,716,413.26

P = Monthly withdrawal = ?

r = Monthly interest rate = 7.8% ÷ 12 = 0.65%, or 0.0065

n = number of months = 20 years * 12 months = 240

Substitute the values into equation (3) and solve for P to have:

PV = P × [{1 - [1 ÷ (1+r)]^n} ÷ r]

$1,716,413.26 = P × [{1 - [1 ÷ (1 + 0.0065)]^240} ÷ 0.0065]

$1,716,413.26 = P × 121.353915567094

P = $1,716,413.26 / 121.353915567094

P = $14,143.86

Therefore, $14,143.86 can be withdrawn each month from the account for 20 years.

For the quarter ended March 31, 2017, Croix Company accumulates the following sales data for its newest guitar, The Edge: $314,000 budget; $300,800 actual. In the second quarter, budgeted sales were $381,000, and actual sales were $391,000.
Prepare a static budget report for the second quarter and for the year to date.
CROIX COMPANY
Sales Budget Report
For the Quarter Ended June 30, 2017
Second Quarter Year to Date
Product Line Budget Actual Difference Budget Actual Difference

Answers

Answer:

CROIX COMPANY

Sales Budget Report

For the Quarter Ended June 30, 2017

                                  Second Quarter                         Year to Date

Product Line  Budget     Actual   Difference    Budget    Actual    Difference

The Edge     381,000  391,000  10,000         695,000   691,800    3,200  U

Explanation:

Croix's sales budget gives a forecast of the sales figure over the future period in order to help Croix plan its production or purchase of the newest guitar, The Edge so that customers' demand can be met and profit objectives of the company is achieved.

1.1. Which of the following ratios are key components in measuring a company's operating efficiency? (You may select more than one answer. Single click the box with the question mark to produce a check mark for a correct answer and double click the box with the question mark to empty the box for a wrong answer.)
a. Profit margin
b. Equity ratio
c. Return on total assets
d. Total asset turnover
1.2. Which ratio summarizes the components applicable in 11?
a. Debt ratio
b. Profit margin
c. Return on total assets
d. Total asset turnover
2. What measure reflects the difference between current assets and current liabilities?
a. Gross margin
b. Day's sales uncollected
c. Retun on total assets
3. Which of the following short-term liquidity ratios measure how frequently a company collects its accounts? (You may select more than one answer. Single click the box with the question mark to produce a check mark for a correct answer and double click the box with the question mark to empty the box for a wrong answer.)
a. Days' sales uncollected
b. Days' sales in inventory
c. Accounts receivable turnover
d. Acid test rato

Answers

Answer:

 1.1 The ratio from the list below which measures the efficiency of the operations of a company is D - Total Asset Turnover Ratio.  

Explanation:

Total Asset Turn Over Ratio is calculated by dividing Net Sales by Average Total Assets.

For example, if company CDH is reporting a value of $499,650 as initial total assets and $387,656 as ending total assets. Within the same period, the company generated sales of $250,655, with sales returns of $17,000.

This means that, the asset turnover ratio for Company CDH is calculated as follows:

($250,655-$17,000)/(($387,656+$499,650)/2)

The answer is 0.52667

Thus, every dollar in total assets generates $0.52667 in sales.

Efficiency ratios are important for rating the operations of the business. They are also used by investors and lenders when conducting financial analysis of businesses to decide whether the companies are a good investment.

1.2 The component which summarises the components applicable in 1.1 is D Total Asset Turnover          

2. Working capital is the variance between current assets and current liabilities.

. This is simply the capital that an organisation uses in its day-to-day business operations.

3.   The short-term liquidity ratios which calculate how frequently a company collects its accounts are:

A) Days' sales Uncollected and

C) Accounts receivable turnover.

A) Days' sales Uncollected is calculated by

(Accounts receivable/Net annual credit sales) x 365

It is the number of days before receivables are collected.

The lower the ratio the more liquid the company is likely to be. High Days' Sales Uncollected Ratios are bad for business.

C) Accounts receivable turnover is the annual rate at which a business collects its average accounts receivable.

Cheers!

All else being equal, a marketing channel that has a high cost per exposure will have a ________ return on investment.

Answers

Answer:

all else being equal, a marketing channel that has a high cost per exposure will have a low return on investment

Juice Drinks has beginning inventory of $10,000, purchases in the amount of $150,000, and ending inventory of $8,000. Juice Drinks cost of goods sold is $ ____________.

Answers

Answer:

$152,000

Explanation:

Given the data as shown below;

Opening inventory = $10,000

Purchases = $150,000

Ending inventory = $8,000

Therefore,

Juice drinks cost of goods sold = Opening inventory + Purchases - Ending inventory

= $10,000 + $150,000 - $8,000

= $152,000

Angie Pereira and Ferro Schwartz are employees of Free Star, Inc. In February 2019. Angie's gross pay was $6000, and Ferro's gross pay was $7400. All earnings are subject to FICA-OASDI Tax of 6.296 and FICA--Medicare Tax of 1.4596. Which of the following would be included in the entry to record the salaries expense for February?
A. a credit to Salaries Expense for 5830.80
B. a credit to FICA-OASDI Taxes Payable for $830.80
C. a debit to FICA-Medicare Taxes Payable for $830.80
D. a debit to Salaries Payable to employees for $830.80

Answers

Answer:

B. a credit to FICA-OASDI Taxes Payable for $830.80

Explanation:

Free Star, Inc. In February 2019.

Angie's gross pay was $6000,

Ferro's gross pay was $7400

Total gross pay $ 13400

FICA-OASDI Tax  6.296 %

$ 13400 * 6.2% = $ 830.80

The recording of the journal entry would require a debit to FICA tax and credit to FICA tax payable .

The FICa tax is 6.2 % which equals to $ 830.80 of the two gross pays.

All the other three options are incorrect.

Logan Corporation issued $800,000 of 8% bonds on October 1, 2006, due on October 1, 2011. The interest is to be paid twice a year on April 1 and October 1. The bonds were sold to yield 10% effective annual interest. Logan Corporation closes its books annually on December 31.

Instructions

(a) Prepare the amortization schedule (effective interest method) through October 1, 2007.

(b) Prepare the adjusting entry for December 31, 2007. Use the effective-interest method.

(c) Compute the interest expense to be reported in the income statement for the year ended December 31, 2007.

Answers

Answer:

a)

period     interest       interest       discount     amortized      bond's

               payment     expense     on BP          discount        carrying value

0                                                     49,320.60                        750,679.40

1               32,000       37,533.97   43,786.63   5,533.97       756,213.37

2              32,000       37,810.67    37,975.96   5,810.67       762,024.04

3              32,000       38,101.20    31,874.76     6,101.20       768,125.24

4              32,000       38,406.26   43,786.63   6,406.26      774,531.50

b)

December 31, 2017, accrued interest on bonds payable

Dr Interest expense 19,050.60

    Cr Interest payable 16,000

    Cr Discount on bonds payable 3,050.60

c)

total interest expense year 2007:

($37,533.97/2) + $37,810.67 + ($38,101.20/2) = $18,776.99 + $37,810.67 + $19,050.60 = $75,638.26

Explanation:

the market price of the bonds:

$800,000 / 1.05¹⁰ = $491,130.60

$32,000 x 8.1109 (PV annuity factor, 4%, 10 periods) = $259,548.80

market price = $750,679.40

discount on bonds payable $49,320.60

discount amortization first payment = (750,679.40 x 0.05) - 32,000 = 5,533.97

discount amortization second payment = (756,213.37 x 0.05) - 32,000 = 5,810.67

discount amortization third payment = (762,024.04 x 0.05) - 32,000 = 6,101.20

discount amortization fourth payment = (768,125.24 x 0.05) - 32,000 = 6,406.26

Your supervisor instructs you to purchase 480 pens and 6 staplers for the workplace. Pens are purchased in sets of 6 for $2.45. Staplers are sold in sets of 2 for $14.95. How much will the purchase of these products cost?

Answers

Answer:

Total cost= $225.9

Explanation:

Giving the following information:

Your supervisor instructs you to purchase 480 pens and 6 staplers for the workplace.

Pens are purchased in sets of 6 for $2.45.

Staplers are sold in sets of 2 for $14.95.

First, we need to calculate the number of "packs" to buy:

Pens= 480/6= 80

Staplers= 6/2= 3

Total cost= 80*2.45 + 2*14.95= $225.9

In an Oligopoly industry a change in price by one firm will _____ impact the other firms in the industry.

Answers

Answer:

The answer is significantly.

Explanation:

Oligopoly is a market situation in which there are few sellers, selling similar goods and services and many buyers. The barriers to entry in this market in high. Example of a oligopoly market is OPEC.

The competition amongst the few sellers is high because they are selling the same thing and a change in price by one firm will significantly affect other firms in the industry. For example, if a firm reduces the price of its goods, this creates a price war and other firms to start reducing their price to match the lower price. And if another firm increases its price, consumers will switch to competitors

Suppose you observe the following situation: Security Beta Expected Return Pete Corp. 1.80 .190 Repete Co. 1.49 .163 a. Assume these securities are correctly priced. Based on the CAPM, what is the expected return on the market? (Do not round intermediate calculations and enter your answer as a percent rounded to 2 decimal places, e.g., 32.16.) b. What is the risk-free rate? (Do not round intermediate calculations and enter your answer as a percent rounded to 2 decimal places, e.g., 32.16.)

Answers

Answer:

a. 12.03%

b. 3.32%

Explanation:

The computation is shown below:

As we know that

Expected rate of return = Risk free rate of return + [Beta × Risk premium]

Let us assume Risk free rate of return be x and Risk premium be y

Now the equations are as follows

For Pete Corp

19 = x + 1.80y ...................... (1)

For Repete Corp

16.3 = x + 1.49y .....................(2)

Now Solving (1) and (2)

After solving we get

y = 8.70967741935

x = 3.3225806452

i.e Risk free rate = x = 3.32%

And, the Risk premium = 8.70967741935%

So,

Expected return on market = Risk free rate + Risk premium

= 3.3225806452 + 8.70967741935

= 12.03%

The following standards for variable manufacturing overhead have been established for a company that makes only one product: Standard hours per unit of output 5.2 hours Standard variable overhead rate $11.60 per hour The following data pertain to operations for the last month: Actual hours 2,500 hours Actual total variable manufacturing overhead cost $29,590 Actual output 150 units What is the variable overhead efficiency variance for the month?

Answers

Answer:

Variable overhead efficiency variance= $19,952 unfavorable

Explanation:

Giving the following information:

Standard hours per unit of output 5.2 hours

Standard variable overhead rate $11.60 per hour

Actual hours 2,500 hours

Actual output of 150 units

To calculate the variable overhead efficiency variance, we need to use the following formula:

Variable overhead efficiency variance= (Standard Quantity - Actual Quantity)*Standard rate

Standard quantity= 5.2*150= 780

Variable overhead efficiency variance= (780 - 2,500)*11.6

Variable overhead efficiency variance= $19,952 unfavorable

Discount stores that try to keep prices as low as possible are more likely to function using ________ operations.

Answers

Answer: self service

Explanation:

Discount stores that try to keep prices as low as possible are more likely to function using self service operations.

Self-Service Operations is quite a straightforward concept whereby the individuals will have to serve themselves. An example of such is discount houses that deals with clothing.

Ideally, in effective marketing planning, goals should be _____ in terms of what is to be accomplished and when.

Answers

Answer:

The answer is quantified and measurable.

Explanation:

Goals need to be quantified and measurable in effective marketing planning. To determine what needs to be accomplished and when, we must put figures to it. This makes performance measurement easier where variances at the end can be analysed.

For example, one of the marketing goals for bank A might be to onboard 100 new customers every month for a year after the launching of its new mobile app.

This example is quantified and can be measured every month.

Facial Cosmetics provides plastic surgery primarily to hide the appearance of unwanted scars and other blemishes. During 2021, the company provides services of $402,000 on account. Of this amount, $52,000 remains uncollected at the end of the year. An aging schedule as of December 31, 2021, is provided below.
Age Group Amount Estimated Percent
Receivable Uncollectible
Not yet due $ 32,000 4 %
0-30 days past due 10,200 6 %
31–60 days past due 7,200 12 %
More than 60 days past due 2,600 30 %
Total $ 52,000
Required:
1. Calculate the allowance for uncollectible accounts.
2. Record the December 31, 2021, adjustment, assuming the balance of Allowance for Uncollectible Accounts before adjustment is $400 (debit).
3. On April 3, 2022, a customer’s account balance of $500 is written off as uncollectible. Record the write-off.
4. On July 17, 2022, the customer whose account was written off in requirement 3 unexpectedly pays $100 of the amount but does not expect to pay any additional amounts. Record the cash collection.

Answers

Answer: Please see explanation for answers

Explanation:

Age Group            Amount           Estimated Percent     Estimated  Amount

                              Receivable      Uncollectible                 Uncollectible

Not yet due             $ 32,000              4 %                          $1,280

0-30 days past due 10,200                 6 %                          $612  

31–60 days past due 7,200                 12 %                        $864

More than 60 days past due 2,600      30 %                      $780

Total                                  $ 52,000                                    $3536

Calculation

1) Estimated Amount Uncollectible = Amount Receivable x Estimated Percent      Uncollectible    =

4% x 32,000= $1,280

6% x 10,200=$612

12% x 7,200=$864

30% x2600=$780

Total = $3,536

The allowance for uncollectible accounts = $3,536

2) Journal to  Record the December 31, 2021, adjustment for a debit of $400

Estimated Amount Uncollectible =$3,536

Adjusted = $3536 + debit $400=$3,936

Date                   Account                  Debit             Credit

Dec 31, 2021,  Bad debts Expense    $3,936

Allowance for uncollectible accounts                    $3,936

3) Journal to  Record the write-off of $500

Date                   Account                              Debit             Credit

April 3, 2022,  Allowance for uncollectible

                             accounts                             $500

                         Accounts receivable                                     $500

4a)Journal to  reinstate  the account previously wrtten off  On July 17, 2022

Date                   Account                              Debit             Credit

July 17, 2022,   Accounts receivable             $100

Allowance for uncollectible  accounts                             $100

4b)Journal to record collection of cash  

Date                   Account                              Debit             Credit

July 17, 2022,   Cash                                    $100

     Accounts receivable                                                     $100

                                                                                                                   

A Journal Entry refers to simply a summary of the debits and also credits of the transaction entry to the Journal. When A Journal entries are important to the transaction because they allow us to sort our transactions into manageable data.

Age Group            Amount         Estimated Percent     Estimated  Amount

                            Receivable     Uncollectible                 Uncollectible

Not yet due             $ 32,000               4 %                          $1,280

0-30 days past due  10,200                   6 %                          $612  

31–60 days past due 7,200                  12 %                        $864

More than 60 days past due 2,600       30 %                      $780

                                                                                                             

Total                                 $ 52,000                                   $3536

The formula apply  Then we Estimated the Amount Uncollectible is =

Amount Receivable x Estimated Percent *Uncollectible   =

4% x 32,000=                 $1,280

6% x 10,200=                   $612

12% x 7,200=                   $864

30% x2600=                    $780

Then the Total is =          $3,536

The allowance for uncollectible accounts = $3,536

                                                                                                                       

Journal Entry

2) Journal to  Record the December 31, 2021, adjustment for a debit of $400

Estimated Amount Uncollectible =$3,536

Adjusted = $3536 + debit $400=$3,936

Date                   Account                  Debit             Credit

Dec 31, 2021,  Bad debts Expense    $3,936

Allowance for uncollectible accounts                    $3,936

                                                                                                         

3) Journal to  Record the write-off of $500

Date                   Account                             Debit            Credit

April 3, 2022,  Allowance for uncollectible

                            accounts                             $500

                        Accounts receivable                                     $500

4a)Journal to  reinstate  the account previously written off  On July 17, 2022

Date                   Account                              Debit             Credit

July 17, 2022,   Accounts receivable             $100

Allowance for uncollectible  accounts                            $100

                                                                                                                 

4b)Journal entry to the record collection of cash  

Date                   Account                              Debit             Credit

July 17, 2022,   Cash                                    $100

    Accounts receivable                                                     $100

Find out more information about Journal entry here:

https://brainly.com/question/8913038

A simple random sample of 20 observations is derived from a normally distributed population with a known standard deviation of 3.2. You may find it useful to reference the z table.
a. Is the condition that X−X− is normally distributed satisfied?
Yes
No
b. Compute the margin of error with 95% confidence. (Round intermediate calculations to at least 4 decimal places. Round "z" value to 3 decimal places and final answer to 2 decimal places.)
c. Compute the margin of error with 90% confidence. (Round intermediate calculations to at least 4 decimal places. Round "z" value to 3 decimal places and final answer to 2 decimal places.)
d. Which of the two margins of error will lead to a wider interval?
The margin of error with 95% confidence.
The margin of error with 90% confidence.

Answers

Answer:

1. It is satisfied

2. 1.4

3. 1.18

4. 95% confidence is wider

Explanation:

1. It is normally distributed since n<30

2. Margin of error with 95% confidence

= Alpha = 1 - 0.95

= O.05

Alpha/2 = 0.025

Z(0.025) = 1.960

Margin of error = z(1.960)*SD/√n

= 1.960*(3.2/√20)

= 1.960 x 0.7156

= 1.4025

Approximately 1.4

3. At 90%

Alpha = 1 -0.9

= 0.10

Alpha/2 = 0.05

Z(0.05) =1.645

E = 1.645 x 3.2/√20

= 1.645 x 0.7176

= 1.177

Approximately 1.18

4. From the calculations in 2 and 3 it is obvious that the margin of error with 95% confidence interval is wider.

A division of a manufacturing company has a return on investment of 24%. The division has an opportunity to accept a project that is expected to earn a return on investment of 22%. The company’s hurdle rate is 20% which of the following statements is true?
a) A division reports the following figures: Profit margin =20% Investment turnover = 0.5. The division return on investment is
b) If a company has $2,000,000 invested in buildings, equipment, and other assets and desires to earn a return on investment of 30%, the company will need to earn a net income of $ .

Answers

Answer:

Return on Investment

The statement that is true is:

b) If a company has $2,000,000 invested in buildings, equipment, and other assets and desires to earn a return on investment of 30%, the company will need to earn a net income of $600,000 (30% of $2,000,000).

Explanation:

The company's Return on Investment is a financial performance measure that calculates the efficiency of the use of investment resources by dividing the returns generated by an investment by the cost of the investment during a period of time.  It can be used to evaluate a divisional manager's performance based on the returns generated from the investments made in the division.

Suppose that Mexico experienced a very severe period of inflation in 1972. As prices in Mexico rose, the demand in the foreign exchange market for Mexican pesos:

Answers

Answer:

demand for pesos would fall and supply would rise. their value would decrease as a result

Explanation:

Inflation is a persistent rise in general price level.

When there is high inflation in a country, the demand for the currency would fall because the value of the currency is low. this fall in demand coupled with the excess supply of the currency would lead to a fall in the value of the currency.

The following data is given for the Bahia Company: Budgeted production 1,049 units Actual production 971 units Materials: Standard price per pound $1.971 Standard pounds per completed unit 12 Actual pounds purchased and used in production 11,302 Actual price paid for materials $23,169 Labor: Standard hourly labor rate $15.00 per hour Standard hours allowed per completed unit 4.3 Actual labor hours worked 5,000.65 Actual total labor costs $76,260 Overhead: Actual and budgeted fixed overhead $1,014,000 Standard variable overhead rate $27.00 per standard labor hour Actual variable overhead costs $140,018 Overhead is applied on standard labor hours. The variable factory overhead controllable variance is a.$75,397.52 unfavorable b.$75,397.52 favorable c.$27,284.90 unfavorable d.$27,284.90 favorable

Answers

Answer:

c.$27,284.90 unfavorable

Explanation:

Standard variable overhead rate                     =$27.00

Standard hours allowed per completed unit  =4.3

Actual production unit                                      =971

Actual variable overhead costs                       =$140,018

Variable factory overhead controllable variance = (Standard variable overhead rate * Standard hours allowed per completed unit * Actual production unit) - Actual variable overhead costs

Variable factory overhead controllable variance = ($27 * 4.3 * 971) - $140,018

Variable factory overhead controllable variance = $112,733.1 - $140,018

Variable factory overhead controllable variance = $27,284.9 (Unfavorable)

Denver Company, a calendar year corporation, had the following actual income before income tax expense and estimated effective annual income tax rates for the first two quarters in year x8: quarter income before tax estimated tax rate first $100k 30% second $140k 24% Denver's income tax expense in its interim income statement for the second quarter should be:

Answers

Answer:

Denver Company

Income Tax Expense for the second quarter:

Pre-tax quarter income = $140,000

Estimated tax rate = 24%

Tax Expense = $140,000 x 24%

= $33,600

Explanation:

a) Data:

Quarter    income before tax        estimated tax rate

first                 $100k                          30%

second           $140k                          24%

b) Denver's quarter second income tax expense is the product of the pretax income for the second quarter and the estimated income tax rate for the quarter.  The resulting calculation shows the estimated income tax expense that has to be settled by Denver.  If it is not settled in the quarter second period, it has to be carried forward to the next quarter as a liability under the heading, Income Tax Payable.

After reading it write about whether or not you agree with the academic economic consensus that independent officials running the Federal Reserve are able to properly balance their dual mandate in a fair and balanced fashion with the needs of workers in one hand and the financial industry on the other. If you agree with the consensus view explain your reasons; or if you disagree and think that the officials are biased in favor of the financial industry explain your reasoning with some possible solutions to the problem. Write at least two paragraphs articulating your views.

Answers

Answer:

The Federal Reserve has been at times biased in favor of the financial industry, because they have often put inflation targeting above the need to reduce unemployment when executing monetary policy. Besides, the financial industry has often been rescued by massive loans from the Fed.

However, the Federal Reserve has also acted in favor of reducing unemployment, specially during recessions, by expanding the money supply through a policy known as quantitative easing.

In conclusion, we can say that the Fed tends to be biased in favor of the financial industry, but not at all times.

Other Questions
What is the scale factor of this dilation? What are half reaction Lila is camping with her family. She wants to hike to the lake, go fishing, and hike back before 6:05 P.M. It will take 1 hour and 10 minutes to hike to the lake and 1 hour and 50 minutes to hike back. Lila wants to fish for 3 hours and 10 minutes. What is the latest time Lila can start the hike to the lake? Find the coordinates of point X that lies along the directed line segment from Y(-8, 8) to T(-15, -13) and partitions the segment in the ratio of 5:2. A. (-5, -15) B. (-23, -5) C. (-13, -7) D. (-11.5, -2.5) Scenario B You are a principal who is trying to figure out the truth about a lunchroom fight. The fight was between Justin and Max. Justin is a new student. He is shy and doesnt have many friends. Max is a popular student who is known for his friendliness. Account A: Justin Max started it. I was just standing in line waiting to pay for my food, and he shoved me super hard. And for no reason! He just freaked out on me. I dont even know the kid, and hes been weird to me ever since I started going to this school. Him and his friends glare at me in English class for no reason. Account B: Max That kid is psycho. He turned around and punched me out of nowhere. Me and my friends were standing in line just joking around, and he turned around and punched me for no reason. Hes messed up and creepy. Ask anyone. Account C: Jamie (student who has class with Max and Justin right before lunch) I wasnt in the cafeteria today, and Im not friends with any of those guys, but Ive seen Max and his friends be mean to Justin in the hallways and in class when the teacher isnt looking. Not physical or anything, but theyll like say jokes under their breath and then laugh and stuff like that. They make him uncomfortable." Kela Corporation reports net income of $450,000 that includes depreciation expense of $70,000. Also, cash of $50,000 was borrowed on a 5-year note payable. Based on this data, total cash inflows from operating activities are: how many are 1 raised to 2 ??? A firm has current assets of $36,000, cash of $5,000, current liabilities of $20,000, total assets of $80,000 and total liabilities of $45,000. What is its net working capital? a. $16,000 b. $28,000 c. $35,000 d. $44,000 will rate you brainliest need help Kim is earning money for a trip. She has saved and she earns per hour babysitting. The total amount of money earned (y) after (x) number of hours worked is given by the equation . How many hours will she need to work in order to earn for her trip? List at least 4 places/content that most often inspire artists. Explain which inspires and draws you the most and why? if you could be any animal what would you be and why 11. Find the slope of the following coordinate points: (11,4)and (5.-3) Given that f(x)=2x+1 and g(x)=-5x+2 slice for f(g(x)) when x=3? find the interest rate r when p = 800, a = 2700, and t = 3. Which of the following is equivalent to 2i(6 7i)? Define enlightenment A. found of Buddhism B. wisdom C. the Buddha's word for release from selfishness and pain D. Nirvana How does the middle row of panels help develop the central idea of this page? How could the notion of maximum parsimony lead to an error in establishing an evolutionary relationship A__________produces finished-goods inventory in advance of customer demand using a forecast of sales.